1. Trang chủ
  2. » Luận Văn - Báo Cáo

lovetoan wordpress com CỰC TRỊ TỔ HỢP

56 3 0

Đang tải... (xem toàn văn)

Tài liệu hạn chế xem trước, để xem đầy đủ mời bạn chọn Tải xuống

THÔNG TIN TÀI LIỆU

Cấu trúc

  • Cực trị trong tổ hợp

    • Bài tập

Nội dung

N gu yễ n T ất T hu Mục lục 1 Cực trị trong tổ hợp 2 I Bài tập 18 1 N gu yễ n T ất T hu 1 CỰC TRỊ TRONG TỔ HỢP §1 Cực trị trong tổ hợp Bài toán cực trị thường xuyên xuất hiện trong các bài toán tổ hợp Để giải quyết bài toán cực trị ta cần vận dụng nhiều nội dung kiến thức liên quan của đại số, tổ hợp Thông thường ta gặp các bài toán sau Bài toán 1 Tìm số nguyên dương k lớn nhất (nhỏ nhất) sao cho với mọi tập con A của tập X nào đó mà |A| = k thì A có tính chất T Để giải quyết bài toán trên, ta t.

Mục lục gu yễ n Tấ t T hu Cực trị tổ hợp I Bài tập 18 N 1 CỰC TRỊ TRONG TỔ HỢP §1 Cực trị tổ hợp Bài toán cực trị thường xuyên xuất toán tổ hợp Để giải toán cực trị ta cần vận dụng nhiều nội dung kiến thức liên quan đại số, tổ hợp Thơng thường ta gặp tốn sau: Bài tốn Tìm số ngun dương k lớn (nhỏ nhất) cho với tập A tập X mà |A| = k A có tính chất T Để giải tốn trên, ta thường theo hai hướng sau: Hướng 1: • Xây dựng tập A0 ⊂ X mà |A0 | = k0 + A0 khơng có tính chất T • Chứng minh tập A X mà |A| = k0 A có tính chất T Khi kmax = k0 Hướng 2: • Ta chứng minh A có tính chất T |A| ≤ k0 hu • Chỉ tập A ⊂ X mà |A| = k0 A có tính chất T T Khi kmax = k0 Tấ t Ví dụ 1.1 Gọi A tập tất số tự nhiên lẻ không chia hết cho nhỏ 30 Tìm số k nhỏ cho tập A gồm k phần tử tồn hai số a, b cho a b N gu yễ n Ta có: A = {1, 3, 7, 9, 11, 13, 17, 19, 21, 23, 27, 29} , |A| = 12 Trước hết ta tập A0 ⊂ A (có số phần tử lớn tốt) mà A0 hai số mà số chia hết cho số Bằng cách thử chọn giá trị ban đầu, ta thấy tập A0 sau có số phần tử nhiều Xét tập A0 = {9, 11, 13, 17, 19, 21, 23, 29} Dễ thấy hai phần tử thuộc A0 khơng chia hết cho Từ ta suy k ≥ Ta chứng minh k = thỏa đề Xét S tập A |S| = Xét ba cặp {21, 7} , {27, 9} , {1, 11}, ta thấy cặp bội Nếu cặp có cặp thuộc S tốn giải Giả sử ba cặp khơng có cặp thuộc S, |S| = nên S phải chữa số cặp chứa số cịn lại Từ suy S phải có cặp {3; 9} {3; 27} cặp bội Hay nói cách khác S tồn hai số chia hết cho Vậy kmin = Nhận xét Mẫu chốt toán phát tập A0 để từ ta khẳng định k ≥ dự đốn kmin = Để tìm tập A0 , ta liệt kê hết số A mà hai số bội Với tốn này, việc tìm tập A0 đơn giản Ví dụ 1.2 (VMO 2004) Cho tập A gồm 16 số nguyên dương Hãy tìm số nguyên dương k nhỏ có tính chất: Trong tập có k phần tử A tồn hai số phân biệt a, b cho a2 + b2 số nguyên tố Giả sử k số nguyên dương cho tập có k phân tử tập A tồn hai số phân biệt a, b cho a2 + b2 số nguyên tố Ta xét tập T gồm số chẵn thuộc tập A Khi |T | = với a, b ∈ T ta có a2 + b2 CỰC TRỊ TRONG TỔ HỢP hợp số, suy k ≥ Xét cặp số sau: A = {1; 4} ∪ {3; 2} ∪ {5; 16} ∪ {6; 15} ∪ {7; 12} ∪ {8; 13} ∪ {9; 10} ∪ {11; 14} Ta thấy tổng bình phương cặp số nguyên tố Xét T tập A |T | = 9, theo ngun lí Dirichlet T chứa cặp nói trên, hay nói cách khác T ln tồn hai số phân biệt a, b cho a2 + b2 số nguyên tố Vậy kmin = Nhận xét a) Vì giả thiết a2 + b2 số nguyên tố nên a2 + b2 số chẵn hay a, b phải khác tính chẵn, lẻ Dựa vào ta xây dựng tập T b) Để tìm phân hoạch tập A thành hợp cặp rời ta làm sau: Ta liệt kê tất số a1 ∈ A,a2 ∈ A, ,a16 ∈ A cho i2 + a2i ( i = 1,16) số nguyên tố Từ ta có phân hoạch trên, phân hoạch T hu Ví dụ 1.3 (VMO 2007) Cho đa giác 2007 đỉnh Tìm số nguyên dương k nhỏ thảo mãn tính chất: Trong cách chọn k đỉnh đa giác, tồn đỉnh tạo thành tứ giác lồi mà số cạnh tứ giác cạnh đa giác cho N gu yễ n Tấ t Gọi đỉnh đa giác A1 ,A2 , ,A2007 Ta thấy tứ giác có đỉnh thuộc đỉnh đa giác có cạnh cạnh cạnh đa giác đỉnh tứ giác đỉnh liên tiếp đa giác Xét tập hợp X = {A1 ,A2 ,A3 ,A4 , ,A2005 ,A2006 }( bỏ đỉnh A4i A2007 ) Ta có |X| = 1505 X không chứa đỉnh liên tiếp đa giác Từ suy k ≥ 1506 Ta chứng minh kmin = 1506 Gọi T tập hợp điểm thuộc đỉnh đa giác |T | = 1506 Ta xét 2007 − 1506 = 501 đỉnh lại Các đỉnh chia đường trịn ngoại tiếp đa giác thành 501 cung, có 2007 + = đỉnh liên tiếp đa giác Dĩ nhiên đỉnh thuộc T cung chứa 501 đỉnh liên tiếp đa giác cho Vậy kmin = 1506 Nhận xét a) Để chứng minh kmin = 1506 ta làm theo cách sau: Đặt T1 = {A1 ,A2 ,A3 ,A4 } , T2 = {A5 ,A6 ,A7 ,A8 } , T501 = {A2001 ,A2002 ,A2003 ,A2004 } ,T502 = {A2005 ,A2006 ,A2007 } Nếu có Ti ⊂ T,i = 1,501 tốn chứng minh Giả sử Ti ⊂ T,i = 1,501, |T | = 1006 nên |T ∩ Ti | = 3, ∀i = 1,501 T502 ⊂ T Vì A2005 ,A2006 ,A2007 ∈ T nên A1 ∈ / T ⇒ A2 ,A3 ,A4 ∈ T ⇒ A5 ∈ / T ta suy A2001 ∈ / T ⇒ A2002 ,A2003 ,A2004 ∈ T Do A2002 ,A2003 ,A2004 ,A2005 ∈ T Bài toán chứng minh b) Mẫu chốt toán đưa nhận xét: Đa giác thỏa yêu cầu toán đỉnh tứ giác đỉnh liên tiếp đa giác Từ xây dựng tập X khơng thỏa u cầu toán Khi xây dựng tập X ta ý, cần xây dựng X cho X không chứa đỉnh liên tiếp X có số phần tử lớn Ví dụ 1.4 Cho m số nguyên dương m ≥ Một tập S gọi “tốt” có m phần tử s1 , s2 , · · · , sm thuộc tập S (không thiết phân biệt) cho s1 +s2 +· · ·+sm−1 = sm Tìm giá trị nhỏ f (m) cho cách phân hoạch tập X = {1; 2; ; f (m)} thành hai tập A, B ln tồn hai tập A B tập “tốt” CỰC TRỊ TRONG TỔ HỢP Với f (m) = m2 − m − 2, ta xét tập X = {1; 2, · · · ; m2 − m − 2} Ta phân hoạch tập X thành hai tập A, B với A = 1; 2, ; m − 2; (m − 1)2 ; (m − 1)2 + 1; ,m2 − m − B = m − 1; m; ; m2 − 2m Xét m − phần tử a1 , a2 , , am ∈ A Nếu a1 ,a2 , ,am−1 thuộc tập {1; 2; ; m − 2} a1 + a2 + · · · + am−1 có giá trị nằm đoạn từ m − đến (m − 1) (m − 2) nên a1 + a2 + · · · + am−1 = am Nếu ∃ai ∈ / {1; 2; · · · ; m − 2} với ≤ i ≤ m − a1 + a2 + · · · + am−1 ≥ (m − 1)2 + m − > m2 − m − hu Do a1 + a2 + · · · + am−1 = am Vậy A không tập tốt Tương tự, với m − phân tử thuộc B tổng chúng không nhỏ (m − 1) (m − 1) = (m − 1)2 > m2 − 2m yễ n Tấ t T Do B khơng tập “tốt” Suy f (m) ≤ m2 − m − khơng thỏa tốn Do f (m) ≥ m2 − m − Xét X = {1; 2; ; m2 − m − 1} = A ∪ B Ta chứng minh hai tập A B có tập “tốt” Giả sử A B không tập “tốt” Không tính tổng quát, ta giả sử ∈ A , suy m − ∈ B (m − 1)2 ∈ A Ta lại có m2 − m − = (m − 1)2 + (m − 2) = (m − 1)2 + + · · · + ∈ B N gu m + m + · · · + m + = m(m − 2) + = (m − 1)2 nên m ∈ B Khi m + m + · · · + m + (m − 1) = m (m − 2) + m − = m2 − m − tổng m − phần tử thuộc B Suy B tập “tốt” Ví dụ 1.5 (IMO Sortlist 2011) Tìm số nguyên dương k lớn cho phân hoạch tập số tự nhiên thành k tập A1 ,A2 , ,Ak cho với số tự nhiên n ≥ 15 i ∈ {1,2, ,k} tồn hai phần tử phân biệt tập Ai có tổng n Xét tập A1 = {1,2,3} ∪ {3m : m ≥ 4} A2 = {4,5,6} ∪ {3m − : m ≥ 4} A2 = {7,8,9} ∪ {3m − : m ≥ 4} Khi đó, tập A1 gồm số tự nhiên n ≥ 13, tập A2 gồm số tự nhiên n ≥ 15, tập A3 gồm số tự nhiên n ≥ 15 Do đó, k = thỏa yêu cầu toán Xét k ≥ 4, giả sử tồn k tập A1 ,A2 , ,Ak thỏa tốn Khi đó, tập A1 ,A2 ,A3 ,A4 ∪ .∪Ak thỏa yêu cầu toán nên ta cần xét k = Đặt Bi = Ai ∩ X, X = {1,2, ,23} với i = 1,2,3,4 Vì số 15,16, ,24 tổng của số thuộc tập Bi Do đó, tập Bi có phần tử Mà |B1 | + |B2 | + |B3 | + |B4 | = 23 CỰC TRỊ TRONG TỔ HỢP nên tồn số j cho |Bj | = Đặt Bj = {x1 ,x2 ,x3 ,x4 ,x5 } Vì tổng hai số khác thuộc vào tập {15,16, ,24} nên ta có (x1 + x2 + x3 + x4 + x5 ) = 15 + 16 + + 24 = 195 Vì 195 không chia hết ta suy điều vơ lí Vậy k = giá trị cần tìm Ví dụ 1.6 Cho tập hợp X = {1; 2; 3; ; 2018} gồm 2018 số nguyên dương A tập tập X thỏa mãn: với x, y, z ∈ A; x < y < z x, y, z độ dài ba cạnh tam giác Hỏi tập hợp A có nhiều phần tử? Giả sử A = {a1 ; a2 ; ; ak } với a1 < a2 < · · · < ak Nếu ak < 2018, đặt t = 2018 − ak bi = + t với i = 1, 2, , k tập {b1 ; b2 ; ; bk } thỏa mãn toán bk = 2018 Do ta giả sử ak = 2018 Ta có a1 + a2 > ak a1 < a2 nên Từ ta có A\ {a1 } ⊂ {1010; 1011; ; 2018}, hu 2a2 > ak = 2018 ⇒ a2 > 1009 ⇒ a2 ≥ 1010 T |A| − ≤ 2018 − 1010 + = 1009 ⇒ |A| ≤ 1010 Tấ t Xét tập hợp A0 = {1009; 1010; ; 2018}, ta có |a0 | = 1010 với x, y, z ∈ A0 mà x < y < z ta có x + y ≥ 1009 + 1010 = 2019 > 2018 ≥ z, yễ n hay x, y, z độ dài ba cạnh tam giác Vậy max |a| = 1010 gu Ví dụ 1.7 Cho bảng (n2 + n + 1) × (n2 + n + 1) Ta điền vào số bảng số số 1, cho khơng có bốn ghi số đỉnh hình chữ nhật Hỏi có nhiều chữ số ghi lên bảng N Gọi xi số số hàng thứ i Ta cần tìm max S với n2 +n+1 S= xi i=1 Gọi tập M gồm cặp (k; l) mà ≤ k < l ≤ n2 + n + 1, ta có |M | = Cn22 +n+1 Với i = 1,2, ,n2 + n + ta xét tập Mi gồm cặp (k; l) mà hai ô giao cột k cột l với hàng thứ i chứa số Ta có |Mi | = Cx2i = xi (xi − 1) Vì khơng có ghi số đỉnh hình chữ nhật nên Mi ∩ Mj = ∅ ∀i = j Do n2 +n+1 |Mi | ≤ |M | i=1 CỰC TRỊ TRONG TỔ HỢP Hay n x2i − xi ≤ i=1 n2 +n+1 n2 +n+1 x2i ⇔ (n2 + n) (n2 + n + 1) i=1 xi ≤ n2 + n − n2 + n + i=1 Mặt khác n2 +n+1 i=1 nên ta có x2i ≥ n +n+1 n2 +n+1 xi i=1 S − S ≤ n2 + n n2 + n + n2 + n + ⇔ S − n2 + n + S − n2 + n n2 + n + ≤0 ⇒ S ≤ (n + 1) n + n + hu Đẳng thức xảy x1 = x2 = · · · = xn2 +n+1 = n + Vậy có nhiều (n + 1) (n2 + n + 1) số ghi lên bảng T Ví dụ 1.8 Cho số nguyên ≤ a1 < a2 < · · · < an = 100 thỏa mãn: Với i ≥ 2, tồn ≤ p ≤ q ≤ r ≤ i − cho Tấ t = ap + aq + ar Tìm giá trị nhỏ giá trị lớn n? N gu yễ n Ta thấy n = 1, nên n ≥ Xét ba số a1 = 20, a2 = 60, a3 = 100, ta có a2 = a1 + a1 + a1 a3 = a2 + a1 + a1 Do n = Ta thấy a1 ≡ (mod 2) a2 = 3a1 ≡ (mod 2) Bằng quy nạp ta có ≡ (mod 2) ∀i = 1,2, , n Dẫn đến an = 100 ≡ (mod 2) (vơ lí) Do a1 ≡ (mod 2) ⇒ a1 ≡ 0, (mod 4) Nếu a1 ≡ (mod 4) chứng minh tương tự ta có an = 100 ≡ (mod 4) (vơ lí) Suy a1 ≡ (mod 4) hay a1 ≡ 0, (mod 8) Chứng minh tương tự ta suy a1 ≡ (mod 8) ≡ (mod 8) ∀i = 1,2, ,n Suy ai+1 − ⇒ ai+1 − ≥ ⇒ ai+1 ≥ + Do 100 = an ≥ a1 + 8(n − 1) ≥ + 8(n − 1) = 8n − ⇒ n ≤ 13 Xét số = 8i − ∀i = 1,2, ,13 ta có ai+1 = + = + a1 + a1 Vậy max n = 13 Ví dụ 1.9 (IMO Shortlist 2012 C2) Cho n ≥ số nguyên dương Xét tập X = {1, 2, 3, , n} gọi A tập cặp phân biệt thuộc X cho tổng cặp khác khơng vượt q n Tìm giá trị lớn |A|? Đặt |A| = x S tổng phần tử tập A, S ≥ + + · · · + 2x = 2x (2x + 1) = x (2x + 1) CỰC TRỊ TRONG TỔ HỢP Mặt khác S ≤ (n + n − 1) + (n − + n − 3) + · · · + n − x + = nx − x (x − 1) Suy x (2x + 1) ≤ nx − 2n − x (x − 1) ⇔x≤ Vì x số nguyên dương nên x≤ 2n − Trước hết ta xét n = 5k + 3, 3k + 3k + 3k 3k + ··· ··· ··· 2n − = 2k + Xét có cặp sau 2k + 2k 4k + 4k + 4k + 4k + 4k + ··· ··· ··· 3k + 2k − 5k + hu Ta chứng minh xmax = 2n − 3k + 2k + 5k + N gu yễ n Tấ t T Các cặp gồm số từ 4k + tổng cặp nhận giá trị từ 3k + đến 5k + 2n − Với trường hợp n = 5k + 4,n = 5k + = 2k + Do đó, chia cặp thỏa toán 2n − = 2k nên cách chia trường hợp n = 5k + ta xóa cặp Với n = 5k + ⇒ (3k + 2,2k + 1) ta 2k cặp thỏa yêu cầu toán Với n = 5k + ta xóa cặp (3k + 2,2k + 1) hàng bảng ta trừ Khi ta thu 2k cặp thỏa toán 2n − Vậy xmax = CỰC TRỊ TRONG TỔ HỢP Ví dụ 1.10 (VMO 2017) Cho số nguyên n > Bảng vng ABCD kích thước n × n gồm n2 ô vuông đơn vị, ô vuông đơn vị tô ba màu: đen, trắng, xám Một cách tô màu gọi đối xứng ô có tâm đường chéo AC tô màu xám cặp ô đối xứng qua AC tô màu đen màu trắng Người ta điền vào ô xám số 0, ô trằng số nguyên dương ô đen mộ số nguyên âm Một cách điền số gọi k− cân đối (với k nguyên dương) thỏa mãn điều kiện sau: i) Mỗi cặp ô đối xứng qua AC điền số nguyên thuộc đoạn [−k; k] ii) Nếu hàng cột giao đen tập số ngun dương điền hàng tập số ngun dương điền cột khơng giao nhau; hàng cột giao ô trắng tập số nguyên âm điền hàng tập số nguyên âm điền cột khơng giao yễ n Tấ t T hu a) Với n = 5, tìm giá trị nhỏ k để tồn cách điền số k− cân đối cho tô màu đối xứng hình bên gu b) Với n = 2017, tìm giá trị nhỏ k để với tô màu đối xứng, tồn điền số k− cân đối N a) Gọi Ai tập số nguyên dương ghi hàng i với i = 1, 2, 3, 4, Theo đề ta có A1 ∩ A3 = ∅ hàng 1, cột giao ô đen cột giống hàng Tương tự A1 ∩ A4 = A3 ∩ A5 = ∅ Ngoài ra, hàng 1, cột có chung trắng góc nên hàng 1, hàng thỏa mãn A1 ∩A5 = ∅ Tương tự A3 ∩A4 = ∅, A4 ∩A5 = ∅ Trước hết, k = khơng đủ khơng thể điền số A1 ∩ A3 = ∅ Giả sử với k = điền được, |A1 | = 1, |A1 | = khơng cịn số để chọn A3 Giả sử A1 = {1}, A3 = A4 = {2} A5 = {1} Nhưng A4 ∩ A5 = ∅ nên dẫn đến mẫu thuẫn Xét k = Ta xét cách điền sau −1 −2 1 2 −1 2 −3 −2 2 3 −3 Vậy kmin = CỰC TRỊ TRONG TỔ HỢP hu b) Trước hết ta xét cách tô màu bàn cờ, tức ô trắng đen xen kẽ hình vẽ, (i,j) tơ màu đen i + j số chẵn tô màu trắng i + j số lẻ T Xét hai ô trắng bảng ô vng vị trí (a; b) (c; d) với ≤ a, b, c, d ≤ 2017 Tấ t • Nếu a + c chẵn b + d chẵn, suy a + d b + c lẻ Khi đó, hai (a; d) (b; c) tơ đen chúng nằm đường chéo màu xám Suy hai ô vuông trắng phải điền số khác yễ n • Nếu a + c lẻ b + d lẻ, xét ô (d; c) điền số với (c; d) rõ ràng ta áp dụng lập luận để suy hai số điền cho hai ô hai khác Từ suy tất số điền cho ô trắng nằm nửa bên phải bảng đơi phân biệt Do đó, ta thu kết gu 20172 − k ≥ + + + · · · + 2016 = 1008 · 1009 = 20172 − thỏa mãn toán quy nạp kết n2 với bảng có kích thước n × n với n số nguyên dương, cụ thể k = Thật vậy, với n = 1, n = 2, n = ta dễ dàng kiểm tra kết tương ứng Xét n ≥ giả sử khẳng định với số nguyên dương bé n Đánh số hàng cột từ → n Ta chứng minh với vị trí đen ln tồn cách điền số ngun dương khơng vượt q kn vào trắng cịn lại bảng (trường hợp điền số âm tương tính bình đẳng) Xét graph G = (V, E) mà V tập hợp đỉnh, đỉnh thứ i ứng với hàng i ≤ i ≤ n; E tập hợp cạnh, có cạnh nối từ đỉnh thứ i đến đỉnh thứ j ô (i; j) ô (j; i) ô màu trắng Ta xét trường hợp sau: N Ta chứng minh k = • Nếu graph G khơng có chứa tam giác, theo định lí Mantel-Turan có khơng n2 n2 n2 q cạnh, nghĩa có khơng q trắng, nên dùng k = số 4 nguyên dương điền vào CỰC TRỊ TRONG TỔ HỢP • Nếu graph G có chứa tam giác, giả sử đỉnh a; b; c phân biệt nối với đôi Điều tương ứng với việc ô (a; b), (b; c), (c; a) (b; a), (c; b), (a; c) giao điểm hàng a; b; c tơ màu trắng Khi đó, số điền vào khơng cần phải phân biệt tập hợp trắng (nếu có) cịn lại hàng a; b; c khơng cần phải rời Rõ ràng hàng cịn lại khơng q n − Khi đó, ta dùng số để điền vào ô trắng dùng không n − số phân biệt để điền vào cịn lại hàng Nếu khơng tính hàng a; b; c, ta lại n − hàng, sử dụng giả thiết quy nạp (n − 3)2 số nguyên dương phân biệt cho hàng cần khơng Do đó, trường hợp này, số lượng số nguyên dương phân biệt cần dùng không vượt 1+n−3+ (n − 3)2 (n − 3)2 + 4(n − 2) (n − 1)2 n2 = = ≤ 4 4 Tóm lại, trường hợp, ta cần sử dụng không hu n2 biệt để điền vào trắng hay nói cách khác k = n × n 20172 − Vậy k = giá trị cần tìm n2 số nguyên dương phân Tấ t T thỏa mãn đề với bảng yễ n Ví dụ 1.11 (P20, Tạp chí Pi, tháng năm 2017) Trong mặt phẳng, xét điểm đôi phân biệt Người ta muốn vẽ đường tròn qua điểm Hỏi vẽ nhiều đường trịn? gu Kí hiệu điểm xét Ai , i = 1,7 Giả sử vẽ n đường tròn, n ∈ N∗ , thỏa mãn điều kiện đề Với i = 1,7, kí hiệu si số đường tròn qua điểm Ai si = 4n Với i = 1,7, gọi mi số cặp đường Do đường tròn qua điểm nên i=1 N tròn mà hai đường tròn cặp qua điểm Ai Đặt m = m= C si i=1 = = ≥ mi , ta có i=1 si − i=2 si i=1 si − 2n i=1 (4n)2 2n(4n − 7) · − 2n = 7 (1) si ) (do si ≥ · i=1 i=1 Mặt khác, đường tròn tùy ý (trong n đường tròn vẽ được) cắt tối đa điểm số điểm Ai nên m ≤ 2Cn2 = n(n − 1) (2) Từ (1) (2), suy 2n (4n − 7) ≤ n(n − 1) (3) 10 CỰC TRỊ TRONG TỔ HỢP Lập luận tương tự ta thu T0 = 12 82 + 22 72 + 32 62 + 42 52 Suy T ≥ T0 với cách viết số Theo bất đẳng thức AM-GM, ta có a+b+c+d+A+B+C +D S≤ 2 ≤ 102 − T0 ≤ 9420 − T = 1022 − T Mặt khác a = 12 , b = 32 , c = 72 , d = 52 , A = 82 , B = 62 , C = 22 , D = 42 , ta có S = 9420 Vậy giá trị lớn S 9420 Câu 1.34 Đáp số toán 1009 Gọi a số câu hỏi mà A hỏi để chắn biết tất số ghi thẻ Ta có nhận xét sau hu ◦ Trong nhóm thẻ hỏi lần hỏi có tối đa thẻ khơng đề cập đến câu hỏi khác Thực vậy, có hai thẻ câu hỏi mà khơng đề cập đến thẻ khác ta biết tập hai số ghi hai thẻ chắn biết số ghi thẻ Tấ t T ◦ Mỗi thẻ phải đề cập câu hỏi Thực vậy, thẻ khơng đề cập đến cho dù bạn A có biết tất số thẻ cịn lại A khơng thể suy số ghi thẻ kia, A tập tất số ghi thẻ yễ n Như vậy, có hai loại thẻ: Loại I gồm thẻ xuất lần câu hỏi Loại II gồm thẻ xuất hai lần câu hỏi Kí hiệu số thẻ Loại I x số thẻ Loại II 2017 − x, ≤ x ≤ 2017 Số lượt thẻ nhắc toàn câu hỏi 3a, ta có 3a ≤ x + × (2017 − x) = 4030 − x N gu Lại nhận xét thứ hai nên x ≤ a Thành thử 3a ≥ 4034 − a hay a ≥ a≤ 4034 = 1008.5 Vậy 4034 = 1009 Thành thử A phải hỏi a0 = 1009 câu hỏi muốn biết tất số ghi thẻ Ta chứng minh có q trình hỏi a0 câu hỏi biết tất số ghi thẻ Xét nhóm thẻ bất kì, tạm đánh số thẻ 1,2,3,4,5,6 Bạn A hỏi Thầy giáo nhóm thẻ {1,2,3}, {3,4,5} {5,6,1} Sau nghe xong câu trả lời A suy số ghi thẻ 3,5,1 chúng thẻ xuất hai lần ba câu hỏi Khi tập hợp số ba thẻ hỏi, A biết tập hợp biết hai số ghi hai thẻ tương ứng Từ A suy số lại ghi hai thẻ Ví dụ nhóm {1,2,3} bạn A biết thẻ 1,3 ghi số biết tập hợp số ghi ba thẻ Do A suy thẻ ghi số ( số ghi thẻ đơi phân biệt) Vì 2017 ≡ ( mod 6) nên ta chia 2017 thẻ cho thành nhóm rời nhau, nhóm 2016 2016 ×3 = = 1008 câu thẻ thẻ dư Theo lập luận A hỏi Thầy hỏi để biết hết tất số ghi thẻ Cộng thêm với thẻ cuối dôi ra, A cần hỏi 1008 + = 1009 câu hỏi rõ tất số ghi thẻ Câu 1.35 Với hai điểm Ai , Aj ta kí hiệu Ai Aj cung Ai kết thúc Aj theo chiều kim đồng hồ kí hiệu m (Ai Aj ) số đo cung Một cung gọi tù 42 CỰC TRỊ TRONG TỔ HỢP m (Ai Aj ) ≥ 180◦ Nhận thấy m (Ai Aj ) + m (Aj Ai ) = 360◦ nên có hai cung tù Kí hiệu xs số cung tù mà hai đầu mút có s − điểm n Nếu s = i có cung Ai Ai+s , Ai+s Ai tù, tổng theo i ta xs + xn−s ≥ n Đẳng thức xảy khơng có đường kính Ai Ai+s Nhận thấy số tam giác không nhọn (tù vuông) số góc khơng nhọn Mỗi cung tù chứa s − điểm có n − s − tam giác không nhọn (dùng hai điểm đầu mút cung kết hợp với điểm cung) Số lượng tam giác không nhọn N = x1 (n − 2) + x2 (n − 3) + · · · + xn−1 + xn−2 Theo bất đẳng thức ta đánh giá được: (s − 1) (xn−s + xs ) ≥ n + + · · · + N≥ s=1 n (n − 1) (n − 3) n lẻ (s − 1) (xn−s + xs )+ s=1 n (n − 2)2 n−2 n−4 n−2 n xn ≥ n + + · · · + = n chẵn + 2 2 Tấ t n−1 N≥ = T n−3 hu n−1 gu yễ n Dấu xảy BĐT không tồn điểm đối xứng qua tâm đường tròn n (n − 1) (n − 2) Số lượng tam giác có đỉnh điểm Cn3 = Vậy số tam giác nhọn N n (n − 1) (n − 2) n (n − 1) (n − 3) (n − 1) n (n + 1) − = n lẻ 24 n (n − 1) (n − 2) n(n − 2)2 (n − 2) n (n + 2) − = n chẵn 24 Câu 1.36 a) Xét tốn tổng qt với bảng vng với kích thước n × n(n ≥ 2) Giả sử ngược lại, hai lân cận có hiệu nhỏ hay n − Với i = 1,2, ,n2 − n, ta đặt Ai = {1,2, ,i},Bi = {i + 1, ,i + n − 1}, Ci = {i + n, ,n2 }, ta nhận thấy số i vng chứa số thuộc Ai không kề với ô chứa số thuộc Ci Vì |B| = n − < n nên với i cố định, tồn dòng cột ghi số thuộc Ai thuộc Ci Với i = tồn dòng cột ghi số thuộc Ci , với i = n2 − n, tồn dòng, cột ghi số thuộc An2 −n Giao dịng cột ghi số Am Cm−1 Điều mâu thuẫn Am ∪ Cm−1 = ∅ Vậy tồn hai số điền vào hai ô lân cận có hiệu lớn hay n = 2017 43 CỰC TRỊ TRONG TỔ HỢP b) Theo câu a k ≥ n = 2017 Xét cách đánh số vào (i,j) số ai,j = (i − 1)n + j,∀i,j ∈ {1,2, ,n} với ai,j số nằm hàng i (từ xuống), cột j (từ trái sang) số số 1,2, ,20172 điền vào ô vuông bảng lần Với ô (i,j) lân cận (i − 1,j); (i + 1,j),(i,j + 1),(i,j − 1) nên số điền vào ô lân cận tương ứng ai−1,j ,ai+1,j ,i,j − 1,i,j + có hiệu khơng 2017 Vậy k = n = 2017 Câu 1.37 Gọi số viết đỉnh hình lập phương a, b, c, d, A, B, C, D Khi S = (b + d + A + C)(a + c + B + D) − T hu với T = aA + bC + cC + dD Ta chứng minh T ≥ 12 82 + 22 72 + 32 62 + 42 52 Vì tập giá trị T hữu hạn nên đạt giá trị nhỏ T0 Với cách viết ta có T0 = a0 A0 + b0 B0 + c0 C0 + d0 D0 T Khơng tính tổng qt, giả sử a0 = 82 Ta thấy A0 = 1, A0 > 1, ta giả sử B0 = 1, Tấ t T0 = 82 A0 + b0 + c0 C0 + d0 D0 > 82 + A0 b0 + c0 C0 + d0 D0 (vơ lí) Lập luận tương tự ta thu T0 = 12 82 + 22 72 + 32 62 + 42 52 Suy T ≥ T0 với cách viết số Theo bất đẳng thức AM-GM, ta có yễ n a+b+c+d+A+B+C +D 2 ≤ 102 − T0 ≤ 9420 − T = 1022 − T gu S≤ N Mặt khác a = 12 , b = 32 , c = 72 , d = 52 , A = 82 , B = 62 , C = 22 , D = 42 , ta có S = 9420 Vậy giá trị lớn S 9420 Câu 1.38 Đặt n = ak + r với a ∈ Z+ ≤ r < k Ta chứng minh nhận xét sau: Nhận xét Nếu gọi T,S tổng số điền vào bảng ô vng có kích thước r × r (k − r) × (k − r) |T | ≤ t = r2 ,k − r2 |S| ≤ s = (k − r)2 ,k − (k − r)2 Rõ ràng bảng ô vng kích thước r × r bị chứa bảng vng kích thước k × k mà tổng số Gọi T tổng số cịn lại bảng k × k sau loại bảng r × r chứa Ta có T + T = ⇒ |T | = |T | Số lượng ô bảng r × r r2 nên |T | ≤ r2 số lượng cịn lại bảng k × k k − r2 nên |T | ≤ k − r2 Do đó: |T | ≤ t = r2 ,k − r2 Hoàn tồn tương tự với bảng (k − r) × (k − r) Nhận xét Nếu kích thước bảng tăng lên k đơn vị tổng số bảng tăng không t + s 44 CỰC TRỊ TRONG TỔ HỢP Xét hình minh họa sau (với k = 3,r = 2) , từ bảng ban đầu, ta thêm vào phía bên phải tương ứng k hàng k cột: r k−r hu r Tấ t T Để tính tổng số điền vào phần tăng thêm, bao gồm phần ngang phần dọc (giao bảng k × k góc) Ta thấy phần nằm ngang chia thành số bảng vng k × k khơng chồng lên dư lại hình chữ nhật kích thước k × (k − r) cuối Tương tự, phần nằm dọc chia thành số bảng vng k × k khơng chồng lên dư lại hình chữ nhật kích thước (k − r) × k gu yễ n Do đó, để tính tổng số phần thêm vào, trước hết, ta tính tổng số hình chữ nhật khơng chồng lên (theo giả thiết tổng 0) Phần hình vng kích thước (k − r) × (k − r) có màu đỏ giao hình vng cuối dãy ngang đầu dãy dọc, tính lần nên cần phải trừ Cuối cùng, ta cộng số bảng vuông góc có kích thước r × r cịn lại N Nói tóm lại, đặt T tổng số bảng vng k × k góc bên phải S tổng số phần chung bảng (k − r) × (k − r) tổng tăng thêm là: T − S Khi đó, theo nhận xét T − S ≤ t + s Nhận xét chứng minh Trở lại tốn, Ta thấy phần màu xanh r × r ban đầu có tổng số khơng vượt q t nên tổng số bảng sau a lần tăng kích thước (từ r × r lên n × n với n = ak + r) tổng khơng vượt t + a(t + s) Đến đây, ta cách xây dựng bảng thỏa mãn Xét bảng vng k × k góc bên phải bảng n × n, ta điền sau: 45 CỰC TRỊ TRONG TỔ HỢP C A r B D k−r • Điền vào phần A hình vng r × r cho tổng số t (có thể điền t số phần cịn lại 0) • Điền vào phần B hình vng (k − r) × (k − r) cho tổng số −s (có thể điền s số −1 phần cịn lại 0) hu • Điền vào phần C,D tùy ý cho tổng chúng s − t Rõ ràng tồn cách điền giới hạn giá trị t,s T Đến đây, ta thực điền số vào bảng ô vuông kề với cách tuần hồn theo chu kỳ k Nghĩa số điền ô (a,b) giống số điền ô Tấ t (a + k,b),(a,b + k),(a − k,b),(a,b − k) Dễ thấy đó, tất hình vng kích thước k × k có tổng hình vng r × r góc bên trái giống với hình vng r × r góc bên phải, tức tổng t yễ n Do đó, cách điền cho ta bảng có tổng gu t + a(t + s) Câu 1.39 N Vậy giá trị lớn cần tìm t + a(t + s) với t,s,a xác định a) Gọi a1 ,a2 , ,a20 số viên kẹo loại kẹo thứ 1,2, ,20 với ≥ Với loại kẹo thứ i (1 ≤ i ≤ 20), ta đếm số (A,B) mà hai học sinh A,B có loại kẹo Số cần đếm C2ai 20 20 C2ai Khi đó, theo giả thiết, tổng số M hay M = i=1 Áp dụng bất đẳng thức Bu-nhi-a-cốp-ki ta có 20 M= i=1 (ai − 1) = i=1 a2i − 20 i=1 2 20 ≥ 20 i=1 · 20 20 − i=1 20202 2020 = − = 101000 2 · 20 Dấu "=" xảy = 101,∀i = 1,2, ,20 Vậy giá trị nhỏ M 101000 46 = 2020 i=1 CỰC TRỊ TRONG TỔ HỢP 19 b) Như lý luận câu a, ta có M = i=1 (ai − 1) = 2 19 a2i − i=1 19 19 i=1 a2i đạt giá trị nhỏ Nên biểu thức M đạt giá trị nhỏ i=1 19 a2i đạt giá trị nhỏ |ai − aj | ≤ với ≤ i,j ≤ 19 Ta chứng minh (1) i=1 Thật Xét số a, b, c, d mà a ≥ b + 2; c = a − 1; d = b + ta có cd = ab + a − b − ≥ ab (a + b)2 = (c + d)2 suy a2 + b2 ≥ c2 + d2 Mở rộng tính chất cho nhiều số ta suy (1) chứng minh Do M đạt giá trị nhỏ có t số có giá trị k 19 − t số có giá trị k + với ≤ t ≤ 19 giá trị nhỏ M= tk + (19 − t)(k + 1)2 − 2020 T hu Ta có tk + (19 − t)(k + 1) = 2020 ⇔ t = 19k − 2001 2001 2020 Do ≤ t ≤ 19 nên ≤k≤ 19 19 Từ ta có k = 106, t = 13 Thay vào ta giá trị nhỏ M [13 · 1062 + · 1072 − 2020] = 106371 N gu yễ n Tấ t Câu 1.40 Ta chứng minh nmax = 66 Giả sử nmax > 66 Cách Khi tồn đại biểu A100 quen 67 đại biểu A1 ,A2 , ,A67 Gọi X tập hợp đại biểu A68 ,A69 , ,A99 |X| = 32 Khi đại biểu A1 ,A2 , ,A67 quen A100 đại biểu thuộc X (vì đại biểu Ai ,Aj với ≤ i < j ≤ 67 quen nhau) Suy họ quen không 33 đại biểu Vì đại biểu quen 34, 35, , 65, 66 đại biểu khác phải thuộc X Điều vô lý |X| = 32 < 66 − 34 + Cách Xét người A có 67 người quen Xét người B có nhiều 34 người quen Nếu A quen B Khi 98 người cịn lại có 66 người quen A hiều 33 người quen B Do 66 + 33 = 99 > 98 nên A, B phải có người quen chung, mâu thuẫn Vậy A khơng quen B Vì có 33 người có số người quen tương ứng 34, 35, , 66 nên có 33 người khơng quen A Điều mâu thuẫn A quen với 67 người Vậy nmax ≥ 66 Ta xây dựng ví dụ thỏa mãn n = 66 sau A1 quen với B1 , B2 , , B66 , A2 quen với B2 , B3 , , B66 , , A34 quen với B34 , B66 Khi khơng có người đôi quen Ak quen với 67 − k người (∀k = 1, ,34) Bk quen với k người (∀k = 1, ,34) Bk quen 34 người với (∀k = 35, ,66) Rõ ràng trường hợp thỏa mãn yêu cầu đề Câu 1.41 Viết số từ đến 20182 vào bảng ô vuông 2018 × 2018 sau: 2019 ··· ··· 2020 ··· ··· 2021 ··· ··· 47 ··· ··· ··· ··· 2017 4035 ··· ··· 2018 4036 ··· 20182 CỰC TRỊ TRONG TỔ HỢP Khi ta thấy số thỏa mãn đỉnh tứ giác đẹp phải bốn ô vuông liên tiếp cột hàng bảng Ta gọi số "đẹp" Vậy để tìm số "đẹp" ta lát bảng vng 2018 × 2018 bảng vng × (1 hàng, cột) × (4 hàng, cột), gọi chung quân domino, cho khơng có hai qn domino có vng chung điểm thuộc tập A thuộc tứ giác nên số thuộc đẹp Ta đếm số quân domino tối đa dùng, hiển nhiên số quân domino tối đa không vượt 20182 : = 10092 Tô đen ô thuộc hàng chẵn cột chẵn bảng ô vuông tô đỏ cịn lại, 2018 2018 × = 10092 số tơ đỏ 2 Để ý quân domino chứa ô tô đỏ số ô tô đỏ lẻ nên ta lát kín bảng vng qn domino trên, tức số quân domino tối đa 10092 Ta cách lát sử dụng 10092 − quân domino gồm hai bước sau: hu • Bước 1: Lát hoàn toàn 2016 hàng bảng với quân domino × 1, hàng liên tiếp ta lát 2018 quân domino ta dùng 2018 × (2016 : 4) = 1009 ì 1008 quõn domino T ã Bc 2: Lỏt hai hàng cuối bảng với quân domino × 4, hàng lát liên tiếp 2018 = 504 trừ hai ô cuối hàng nên ta dùng tới 504 × = 1008 quân domino Tấ t Khi ta dùng 1008 × 1009 + 1008 = 1008 × 1010 = 10092 − quân domino Vậy số tứ giác "đẹp" lớn tạo thành 10092 − yễ n Câu 1.42 Nhận xét: Hai vận động viên tham gia chung không môn thi đương đương với hai môn thi chung khơng q vận động viên Ta đưa việc đếm S có dạng (X,Y,Z) với hai mơn X,Y có vận động viên Z tham gia chung gu • Đếm theo mơn thi đấu, ta có S ≤ C12 N • Đếm theo vận động viên: ta gọi x1 ,x2 , ,xn số môn mà vận động viên 1, 2, ,n tham gia Bằng cách đếm số lượt tham gia ta có x1 + x2 + + xn = 288 Điều dẫn đến S = Cx21 + Cx22 + + Cx2n = (x21 + x22 + + x2n − 288) Qua hai cách đếm ta suy C12 ≥ S = (x21 + x22 + + x2n − 288) hay x21 + x22 + + x2n ≤ 420 Áp dụng bất đẳng thức Cauchy – Schwarz, ta có x21 + x22 + + x2n (x1 + x2 + + xn )2 2882 ≥ = n n 48 CỰC TRỊ TRONG TỔ HỢP Suy 2882 hay n ≥ 198 n 288 = 1, ta dự đoán cực trị xảy Để ý với n ≥ 198 ước lượng n số xi nhận giá trị Do vậy, với i ∈ {1,2, ,n} ta đánh giá 420 ≥ (xi − 1)(xi − 2) ≥ ⇔ x2i ≥ 3xi − 2, ∀i = 1,n Đến đây, ta suy 420 ≥ x21 + x22 + + x2n ≥ (3x1 − 2) + (3x2 − 2) + + (3xn − 2) = 3(x1 + x2 + + xn ) − 2n = 3.288 − 2n hu thu n ≥ 222 Vì bất đẳng thức xảy có 156 số xi 66 số xi nên giá trị nhỏ cần tìm 222 T Câu 1.43 Tổng số mà 100 gấu hái + + · · · + 100 = 5050 (quả) Tấ t Do gấu có số nhau, gấu có nên số sói ăn khơng vượt 5050 − 100 = 4950 (quả) N gu yễ n Ta chứng minh sói có cách “ăn chia” để số ăn 4950 Ta có nhận xét sau: Nhận xét Giả sử có k gấu (k ≥ 2), đặt tên theo thứ tự từ bé đến lớn “1”, “2”, , “k” tương ứng có số 1,2, ,k Khi đó, cách thực quy trình “ăn chia” đây, sói ăn k − làm cho số gấu “1”, “2”, , “k − 1”, “k” tương ứng 1,1,2, ,k − 2,k − 1: Quy trình “ăn chia” sói: Với t = 1,2, ,k − 1, bước thứ t, sói gộp số hai gấu “k − t + 1” “k − t” với nhau, “ăn chia” theo nguyên tắc nêu đề Chứng minh Bằng quy nạp theo t, ≤ t ≤ k − 1, dễ dàng chứng minh khẳng định sau: “Ở bước quy trình, sói ăn sau bước thứ t, số gấu “1”, , “k − t”, “k − t + 1”, , “k” tương ứng 1, ,k − t,k − t, , k − ” Từ khẳng định trên, hiển nhiên suy kết luận Nhận xét Đặt tên 100 gấu, theo thứ tự từ bé đến lớn, “1”, “2”, , “100” Từ nhận xét dễ dàng suy sói thực việc “ăn chia” theo cách sau: • Lần 1: Sói thực quy trình “ăn chia” nêu 100 gấu (k = 100); • Lần 2: Sói thực quy trình cho 99 gấu, từ gấu “2” đến gấu “100”(k = 99); • ··· • Lần thứ m: Sói thực quy trình cho 100 − m + gấu, từ gấu “m” đến gấu “100” (k = 100 − m + 1); • ··· • Lần thứ 99: Sói thực quy trình cho gấu, gấu “99” gấu “100” k = 2) 49 CỰC TRỊ TRONG TỔ HỢP Bằng quy nạp theo m, ≤ m ≤ 99, dễ dàng chứng minh khẳng định sau: “Với cách thực quy trình trên, sau lần thực thứ m, sói ăn số là” 99 + 98 + · · · + (100 − m) + (100 − m + 1) (quả) số 100 gấu “1”, “2”, , “100” tương ứng 1,1, ,1,1,2,3, ,100 − m Từ khẳng định vừa nêu suy ra, sau lần thực quy trình thứ 99, số 100 gấu số sói ăn là: 99 + 98 + · · · + + = 4950 (quả) Vậy, tóm lại, cách thực quy trình “ăn chia” sói ăn tối đa 4950 a a +ai+1 i i = xi+1 Do đó, xi i+1 · xai+2 ai+1 = q ·ai+1 = xi+2 xi+1 yễ n xi+1 xi Tấ t T hu Câu 1.44 Với n số nguyên dương p số nguyên tố, ta kí hiệu vp (n) số lớn số tự nhiên k mà pk |n Dễ thấy vp (mn) = vp (m) + vp (n), với số nguyên dương m, n với số nguyên tố p Giả sử x1 < x2 < · · · < xn số nguyên dương vừa thuộc S vừa số hạng cấp số nhân tăng, có cơng bội q > Vì 1,2,4,8,16 ∈ S 1,2,4,8,16 số hạng cấp số nhân tăng có cơng bội lớn nên giả sử n ≥ Vì x1 < x2 < · · · < xn số hạng cấp số nhân tăng với công bội q nên tồn số nguyên dương a1 , a2 , ,an cho xi+1 = xi · q , xi+1 = q với i = 1,n − Suy ra, với i = 1,2, ,n − 2, ta có với i = 1,n − hay xi với i = 1,2, ,n − Vì vậy, với p số nguyên tố tùy ý, ta có gu (ai + ai+1 ) vp (xi+1 ) = ai+1 vp (xi ) + vp (xi+2 ) N với i = 1,2, ,n − Suy ra, với p số nguyên tố tùy ý, ta có vp (xi ) < vp (xi+1 ) ⇔ vp (xi+1 ) < vp (xi+2 ) với i = 1,n − Dễ thấy, tồn số nguyên tố p cho vp (x1 ) < vp (x2 ) (vì ngược lại, vp (x1 ) > vp (x2 ) với số nguyên tố x2 |x1 , trái với giả thiết x1 < x2 ) Từ đó, suy tồn số nguyên tố p cho vp (x1 ) < vp (x2 ) < · · · < vp (xn ) Do đó, vp (xn ) ≥ n − Suy pn−1 |xn Mà xn ≤ 2017 nên pn−1 ≤ 2017 Vì thế, 2n−1 ≤ 2017 Suy ra, n ≤ 11 Dễ thấy, 1,2,22 , ,210 ∈ S số lập thành cấp số nhân tăng, có cơng bội lớn Vậy, có tối đa 11 phần tử thuộc S số hạng cấp số nhân tăng, có cơng bội lớn Câu 1.45 Dễ thấy, với a số ngun dương, ta có: • Nếu a số chẵn (tức a chia hết cho chia dư 2) a3 chia hết cho 4; (1) • Nếu a số lẻ (tức a chia dư chia dư 3) a3 chia dư (nếu a chia dư 1) a3 chia dư (nếu a chia dư 3) (2) 50 CỰC TRỊ TRONG TỔ HỢP Xét cách chọn số số thỏa mãn yêu cầu đề bài, từ 2017 số nguyên dương Giả sử số chọn n (n ≥ 2) có m số chẵn, k số lẻ Do tổng lập phương số n số chọn không chia hết cho 4, nên từ (1) (2) suy ra, với a, b hai số nguyên dương tùy ý n số đó, ta có a, b khác tính chẵn lẻ, a, b lẻ đồng thời có số dư chia cho Từ đó, 2017 số nguyên dương có 505 số chia dư 504 số chia dư 3, nên suy ta có: • m ≤ (vì m ≥ n số chọn tồn số chẵn); • k ≤ 505 (vì k ≥ 506 n số chọn tồn số lẻ đồng thời có số dư khác chia cho 4) Tấ t ([x] kí hiệu số ngun lớn khơng vượt q x).” T hu Như vậy, n = m + k ≤ + 505 = 506 Dễ thấy, từ 2017 số nguyên dương đầu tiên, chọn 506 số gồm 505 số chia dư số chẵn (tùy ý) ta có 506 số thỏa mãn yêu cầu đề Vậy, chọn tối đa 506 số từ 2017 số nguyên dương cho tổng lập phương hai số số chọn số không chia hết cho Nhận xét Kết quen thuộc sau sử dụng để tìm số số chia dư số số chia dư 3, số 2017 số nguyên dương đầu tiên: “Cho m, k, n số nguyên dương, với ≤ m ≤ k ≤ n Kí hiệu s số số nguyên dương chia k dư m, n số nguyên dương đầu tiên; ta có: n−m+k s= k gu yễ n Câu 1.46 Giả sử điểm tô hai màu, đen trắng Xét điểm A1 , A2 , ,A8 , tô màu sau: A1 , A2 ,A5 ,A − tô màu trắng; A3 , A4 ,A7 ,A8 tơ màu đen Dễ thấy, khơng có ba điểm Ai ,Aj ,A2j−i (1 ≤ i < 2j − i ≤ 8) tơ màu Do n ≥ Ta chứng minh n = số có tính chất đề u cầu Thật vậy, giả sử ngược lại, tồn cách tô điểm A1 , A2 , ,A9 hai màu, đen trắng cho khơng có ba điểm Ai ,Aj ,A2j−i (1 ≤ i < 2j − i ≤ 9) tô màu N Trường hợp 1: Tồn i ∈ {3; 4; 5}, cho điểm Ai Ai+2 tô màu, giả sử màu trắng Khi đó, điểm Ai−2 ,Ai+1 , Ai+4 phải tô màu đen (chú ý i − ≥ i + ≤ 9), vô lý Trường hợp 2: Với i ∈ {3; 4; 5}, điểm Ai Ai+2 tô khác màu Khơng tính tổng qt, giả sử A5 tơ màu trắng Khi đó, A3 A7 tơ màu đen Nhờ tính đối xứng, ta giả sử A4 tô màu trắng A6 tơ màu đen Khi A8 tơ màu trắng (do A6 , A7 A8 không tô màu); suy ra, A2 tô đen (do A2 , A5 A8 không tô màu) A9 tô trắng (do A7 ,A8 A9 không tơ màu) Do đó, A1 phải vừa tô màu trắng (do A1 ,A2 A4 không tô màu),vừa tô đen (do A1 , A5 A9 không tô màu), điều vô lý Những điều vơ lí nhận nhận cho ta điều muốn chứng minh Vậy tóm lại, n = số ngun nhỏ có tính chất đề yêu cầu Nhận xét Những nhược điểm suy luận lôgic thường gặp giải này: - Do không ý thức số ngun n ≥ khơng có tính chất đề yêu cầu số nguyên n − khơng có tính chất đó, học sinh thường cồng kềnh hóa lời giải việc xét số nguyên thuộc tập hợp {3; 4; 5; 6; 7; 8} 51 CỰC TRỊ TRONG TỔ HỢP - Nhiều học sinh chưa hiểu rõ khái niệm giá trị lớn nhất, giá trị nhỏ nên giải toán theo lượt đồ sau: + + + + Chứng minh số n thuộc tập hợp {3; 4; 5; 6; 7; 8} khơng có tính chất đề u cầu; Chứng minh n = số có tính chất đề yêu cầu; Chứng minh tất số ngun n > có tính chất đề yêu cầu; Từ chứng minh bước trên, suy n = số nguyên nhỏ cần tìm hu Có thể phát biểu tốn dạng: “Tìm số ngun n ≥ nhỏ có tính chất: Với cách phân chia tập hợp gồm n số nguyên dương thành hai tập khác rỗng, tồn dãy số cách gồm số hạng mà tất số hạng thuộc tập tập con.” Dạng phát biểu gợi ý cho ta đặt tìm cách giải tốn khái quát sau: Bài toán khái quát Cho số nguyên k ≥ Tìm số nguyên n ≥ k nhỏ có tính chất: Với cách phân chia tập hợp gồm n số nguyên dương thành hai tập khác rỗng, tồn dãy số cách gồm k số hạng mà tất số hạng thuộc tập tập Lưu ý rằng, hướng khái quát yễ n Tấ t T Câu 1.47 Chia tập hợp {1, 2, 3, , 4013, 4014} thành 2007 phần P1 ∪ P2 ∪ ∪ P2007 (mỗi tập hợp chứa phần tử {1, 2, 3, , 4013, 4014}) thỏa mãn tập hợp Pa chứa tất số nguyên dương có dạng 2n (2a − 1), n số khơng âm Khi đó, tập hợp A {1, 2, 3, , 4013, 4014} chứa hai phần tử thuộc 2007 tập hợp khơng rõ ràng có số chia hết cho số lại, mâu thuẫn Mặt khác, A lại có 2007 phần tử nên A chứa phần tử tập Pa nói Gọi αi phần tử A với αi ∈ Pi , i = 1, 2007 Xét phần tử α1 , α2 , α5 , α1094 có dạng 2n , 3.2n , 32 2n , , , 37 2n ; rõ ràng phần tử có hai ước nguyên tố Ta thấy lũy thừa lớn α1 phải lớn lũy thừa lớn α2 ngược lại α2 α1 , mâu thuẫn N gu Hoàn toàn tương tự với phần tử khác dãy α1 , α2 , α5 , α1094 , tức i < j lũy thừa αi phải lớn lũy thừa αj Do đó, lũy thừa dãy α1 , α2 , α5 , α1094 dãy giảm thực Do có phần tử dãy (tương ứng với lũy thừa thay đổi từ đến 7) nên giá trị α1 27 = 128 Hơn nữa, phần tử dãy có giá trị lũy thừa tăng dần (nếu ngược lại, giá trị lũy thừa giảm mà lũy thừa giảm nên có số chia hết cho số khác, mâu thuẫn) Suy ra: α1 giá trị nhỏ dãy Hoàn toàn tương tự với số nguyên tố khác, ta xét số có dạng αi = 2ni 3ni · · · pknk , 2, 3, , pk ước nguyên tố không vượt 4014 Từ suy α1 phần tử nhỏ phần tử tập A Do phần tử nhỏ mA tập A α1 , chứng minh α1 ≥ 128 hay mA ≥ 128 Ta chứng minh 128 giá trị nhỏ mA cách tập hợp A {1, 2, 3, , 4013, 4014} thỏa mãn điều kiện đề Xét tập hợp: A = αi = 2f (i) (2i − 1)|1338 ≤ 3f (i) (2i − 1) ≤ 4014 Rõ ràng tập hợp có 2007 phần tử thuộc {1, 2, 3, , 4013, 4014} Ta chứng minh αx không chia hết cho αy , với x > y (tức tập hợp thỏa mãn đề bài) Thật vậy, giả sử ngược lại tồn x, y thỏa mãn αx αy , đó: 2f (x) (2x − 1) 2f (y) (2y − 1), 52 CỰC TRỊ TRONG TỔ HỢP tức f (x) ≥ f (y) (2x − 1) (2y − 1) Từ cách xác định giá trị u, v ta có: 4(2x − 1) (2y − 1) ⇒ 2x − ≥ 3(2y − 1), đồng thời 43f (y)+1 (2y − 1) ≥ 4014 > 3f (x) (2x − 1) ≥ 3f (x)+1 (2y − 1) ⇒ f (y) > f (x) Mâu thuẫn chứng tỏ tất phần tử A thỏa mãn với a, b ∈ A a khơng chia hết cho b Vậy 128 giá trị nhỏ mA cần tìm Câu 1.48 Trước hết ta thấy rằng: Nếu S tập khuyết T tập S = {n − x|x ∈ S} tập khuyết T Thật vậy: Giả sử ngược lại S khơng phải tập khuyết, tồn hai số nguyên dương s1 , s2 ∈ S cho |s − s | = c n với c số nguyên dương khơng vượt q Khi xét tương ứng hai phần tử s1 = n − s1 ,s2 = n − s2 rõ ràng s1 ,s2 ∈ S tức tồn hai phần tử s1 , s2 ∈ S n T |s1 − s2 | = c ≤ hu |s1 − s2 | = |(n − s ) − (n − s )| = |s − s | = c, N gu yễ n Tấ t S tập khuyết Mâu thuẫn suy nhận xét chứng minh Hơn nữa, |S| = |S | nên S có số phần tử lớn tương ứng có tập S có số phần tử lớn với S n Từ đó, ta thấy xét tập khuyết S có số số phần tử khơng vượt q khơng số n n n số phần tử lớn Xét hai tập hợp sau: A = x | x ∈ S, x ≤ , B = x | x ∈ S, x > 2 A ∩ B = ∅,A ∪ B = S theo cách xác định S |A| ≥ |B| n Khi với c số ngun dương khơng vượt q , ta xét tập hợp: C = {x+c|x ∈ A} Ta có: |A| = |C| Do A ⊂ S nên A tập khuyết rõ ràng A∩C = ∅,B ∩C = ∅ (vì ngược lại tồn hai phần tử thuộc S mà hiệu chúng c, mâu thuẫn) Suy tất phần tử thuộc tập A B C số nguyên dương không vượt n, tức (A ∪ B ∪ C) ⊂ T ⇒ |A| + |B| + |C| ≤ |T | = n Kết hợp điều lại, ta được: |A| + |B| ≤ n Do đó: |A| + |B| ≤ |A| + |B| 2n ≤ 3 Hơn nữa: A ∩ B = ∅, A ∪ B = S |S| số nguyên nên |S| = |A| + |B| ≤ 2n Do số phần tử tập khuyết S T không vượt Ta tập khuyết thỏa mãn đề có 2n 2n phần tử Thật vậy, xét hai tập hợp A, B sau: A= 1,2,3, , S = A ∪ B Chọn c = n+1 , B = n− n+1 n ≤ Ta thấy: 53 n n + 1, n − + 2, ,n 3 CỰC TRỊ TRONG TỔ HỢP n+1 n+1 −1< 3 • Hiệu hai phần tử A khơng vượt q • Hiệu hai phần tử B khơng vượt q n − (n − n n+1 n + 1) = −1< 3 • Hiệu phần tử thuộc B với phần tử thuộc A khơng nhỏ hơn: (n − n n+1 n n+1 n+2 n+1 + 1) − ≥n− − +1= > 3 3 3 Khi đó, rõ ràng S = A ∪ B tập khuyết T ứng với giá trị c = Ta chứng minh |S| = n+1 n ≤ 2n Từ cách xác định A, B, ta có: n n+1 ,B = 3 hu |A| = Ta cần có: T n+1 n 2n + = ,∀n ∈ N 3 Tấ t Xét trường hợp: (∗) • Nếu n chia hết cho 3, tức n có dạng 3m,m ∈ N Suy ra: yễ n n 3m + 3m 6m 2n n+1 + = + = m + m = 2m = = 3 3 3 • Nếu n chia dư 1, tức n có dạng 3m + 1,m ∈ N Suy ra: N gu 3m + 6m + n+1 3m + 6m 2n n = = + + = m + m = 2m = = 3 3 3 • Nếu n chia dư 2, tức n có dạng 3m + 2,m ∈ N Suy ra: n+1 3m + 6m + n 3m + 6m + 2n + = + =m+1+m= = = 3 3 3 Từ suy (*) chứng minh hay tập hợp S cho tập khuyết có Vậy giá trị lớn số phần tử tập khuyết S T 2n 2n Câu 1.49 a) Với m = 4, ta chứng minh số máy bơm nước nhỏ thỏa mãn yêu cầu đề n Điều kiện đủ hiển nhiên với cách đặt cột máy bơm hàng thứ hai sau: X X X ········· ········· ········· ········· 54 X X X CỰC TRỊ TRONG TỔ HỢP Chú ý máy bơm tưới tối đa ô nên điều kiện cần rõ ràng với n = n = Ta chứng minh điều kiện cần phản chứng Giả sử tồn n cho cánh đồng kích thước × n tưới n máy bơm nước Gọi n0 số nguyên dương n nhỏ Theo ý ta phải có n0 ≥ Xét cánh đồng kích thước × n0 Theo định nghĩa n0 , tồn cách xếp k < n0 máy bơm để tưới hết cánh đồng Vì số máy bơm nhỏ số cột nên phải tồn cột khơng chứa máy bơm (ta gọi cột trống) • Ta thấy cột trống cột biên cột trống cột biên, chẳng hạn cột thứ để tưới cột trống, cột thứ hai phải chứa máy bơm Khi đó, cách thêm máy bơm vào cột hàng (nếu chưa có máy bơm), ta thấy n0 − cột lại (bỏ cột 2) tưới k − + = k − < n0 − máy bơm, mâu thuẫn với cách chọn n0 hu • Vì cột trống khơng nằm biên, ta xét cột trống từ bên trái sang Ta giả sử cột cột j Để tưới ô cột trống này, tổng cộng hai cột hai bên cột trống phải có máy bơm (*) Xét trường hợp sau: gu yễ n Tấ t T – Cột j − chứa máy bơm Khi cột từ đến j − trống nên j cột đầu chứa j máy bơm Suy n0 − j cột sau chứa nhiều k − j máy bơm Vì ngăn cách cột trống nên rõ ràng máy bơm bơm cho tất cách đồng kích thước × (n0 − j) Vì k − j < n0 − j nên điều mâu thuẫn với cách chọn n0 – Cột j − chứa máy bơm, đó, (*), cột j + phải chứa máy bơm Khi đó, j + cột đầu chứa j − + + = j + máy bơm nên n0 > k ≥ j + 2, tức bên cạnh cột j + cột Bây giờ, cách thêm vào cột hàng j + máy bơm cần, ta thấy cánh đồng gồm n0 − (j + 1) cột lại sau bỏ j + cột đầu tưới k − (j + 2) + = k − j − máy bơm Vì k − j − < n0 − (j + 1) nên ta nhận mâu thuẫn với cách chọn n0 Như điều kiện cần chứng minh Ta có kết luận: Với cánh đồng × n cần n máy bơm để tưới nước thỏa mãn yêu cầu toán N n−1 Trước hết ta chứng minh điều kiện đủ Với n < điều kiện đủ hiển nhiên, ta xếp cột máy bơm Với n = 5, ta có cách xếp sau: b) Ta chứng minh số máy bơm để tưới cánh đồng × n n − X X X X Từ dễ dàng cách đặt máy bơm cho n Chẳng hạn với n = 20, ta đặt 16 máy bơm sau X X X X X X X X X X X X X X X X Bây ta chứng minh điều kiện cần Chú ý máy bơm nước tưới tối đa ô nên với n = 1, 2, ta thấy cần phải có n máy bơm nước tưới 55 CỰC TRỊ TRONG TỔ HỢP tất ô cánh đồng × n n−1 Đặt f (n) = n − với n = 1,2, Giả sử tồn số nguyên dương n cho cánh đồng kích thước × n tưới k < f (n) máy bơm Gọi n0 số nhỏ Theo ý ta có n0 ≥ Do f (n0 ) ≥ n0 nên ta có k < n0 , có cột trống Lý luận tương tự phần 1, ta thấy cột trống biên Xét cột trống từ bên trái sang Giả sử cột j Khi đó, để tưới ô cột j, hai cột kề bên cột j phải chứa máy bơm nước Xét trường hợp sau: • Cột j − chứa máy bơm nước Khi j cột đầu chứa j máy bơm nước (do cột từ đến j − chứa 1, cột j − chứa 2) Suy n0 − j cột cịn lại chứa khơng q k − j máy bơm nước máy bơm tưới hết cánh đồng kích thước × (n0 − j) Ta có n0 − j − n0 − ≤ n0 − j − = f (n0 − j) 4 hu k − j < f (n0 ) − j = n0 − j − nên từ ta suy điều mâu thuẫn với cách chọn n0 Tấ t T • Cột j − chứa máy bơm, đó, cột j + phải chứa máy bơm Như j + cột đầu chứa j + máy bơm Suy n0 − (j + 1) cột cịn lại chứa khơng k − (j + 1) máy bơm nước Ta xét hai trường hợp sau – Trường hợp 1: Cột j + cột trống Khi n0 − (j + 2) cột lại sau bỏ j + cột đầu tưới đủ nhiều k − (j + 1) máy bơm nước Ta có yễ n k − (j + 1) < f (n0 ) − (j + 1) = n0 − (j + 1) − n0 − n0 − − n0 − (j + 2) − ≤ n0 − (j + 2) − = f (n0 − (j + 2)) gu = n0 − (j + 2) − N (do j ≥ nên j + ≥ 4) Điều mâu thuẫn với cách chọn n0 – Cột j + có máy bơm Khi n0 − (j + 2) cột lại sau bỏ j + cột n0 tưới đủ ô cột (có n0 − (j + 1) cột) Theo tính toán trên, số máy bơm cột khơng q k − (j + 1) Ta lại có đánh giá k − (j + 1) < f (n0 ) − (j + 1) n0 − n0 − (j + 1) − ≤ n0 − (j + 1) − = f (n0 − (j + 1)) = n0 − (j + 1) − mâu thuẫn với cách chọn n0 Bài tốn giải hồn toàn 56

Ngày đăng: 15/04/2022, 19:51

TÀI LIỆU CÙNG NGƯỜI DÙNG

TÀI LIỆU LIÊN QUAN

w